考试课堂中行距问题

考试课堂中行距问题

我正在为 baby rudin 写解决方案手册,经过一番研究,我偶然发现了考试包,它可以帮助我正确地编写问题和答案,但我遇到了一些问题,因为我对 latex 了解不多,所以我逐一介绍。附件:我的代码和结果的屏幕截图,以便更好地理解。

在此处输入图片描述

(1) 如您所见,我使用 documentclass 下的 exam 并编写了所有问题,但与 amsart 相比,行距太大。我可以更改行距,以便两个问题的间距更小吗?

(2) 观察问题 1 和“Ans”之间的距离。每当我写答案时,它都会从新行开始。我怎样才能将其带回到“Ans”的同一行?

(3) 页码看起来像“第 2 页”。 有什么方法可以将其更改为仅“2”。

(4) 我想给这个解决方案手册起个标题。在 amsart 中我按如下方式操作“ \title[]{Algebra} \author[]{Ripan Das} \begin{document} \maketitle \newpage”,但这个东西在这里不起作用。请解决这些问题。我还附上了代码。

\documentclass[answers, 11pt]{exam}


\usepackage{graphicx}
\usepackage{tikz}
\usetikzlibrary{cd,matrix,arrows,decorations.pathmorphing}

\usepackage{
  %amsmath,
  %amsthm,
  amssymb,
  euscript,
  %enumerate,% better enumitem
  url,
  verbatim,
  calc,
}

\textwidth6.5in
\textheight9in
\oddsidemargin.2in
\evensidemargin.2in
\topmargin-1cm
\renewcommand{\baselinestretch}{1.2}
\usepackage{amsmath}
\newtheorem{theorem}{Theorem}[section]
\newtheorem{definition}[theorem]{Definition}%[theorem]
\newtheorem{defn}[theorem]{Definition}
\newtheorem{qns}[theorem]{Question}
%\newtheorem*{qns*}{Question}
\newtheorem{problem}[theorem]{Problem}
%\newtheorem{exercise}[theorem]{Exercise}
\newtheorem{ex}[theorem]{Exercise}
\newtheorem{example}[theorem]{Example}%[section]
\newtheorem{eg}[theorem]{Example}
%\newtheorem*{eg*}{Example}
\newtheorem{obs}[theorem]{Observation}
%\newtheorem{obs*}{Observation}
%\newtheorem*{Obs*}{Observation}
\newtheorem{proposition}[theorem]{Proposition}%[section]
\newtheorem{prop}[theorem]{Proposition}
%\newtheorem{theorem}{Theorem}[section]
\newtheorem{remark}[theorem]{Remark}%[section]
%\newtheorem*{remark*}{Remark}
\newtheorem{corollary}[theorem]{Corollary}%[section]
\newtheorem{lemma}[theorem]{Lemma}%[section]
\newcommand{\QQ}{\mathbb Q}
\newcommand{\ZZ}{\mathbb Z}
\newcommand{\CC}{\mathbb C}
\newcommand{\FF}{\mathbb F}
\newcommand{\RR}{\mathbb R}
\newcommand{\NN}{\mathbb N}
\unframedsolutions
%\operatorname{deg}{{\deg}}
\newcommand\sbullet[1][.5]{\mathbin{\vcenter{\hbox{\scalebox{#1}{$\bullet$}}}}}
\newcommand{\norm}[1]{\left\lVert#1\right\rVert}
\newcommand{\gen}[1]{\langle#1\rangle}
\newcommand{\Hom}[1]{\text{Hom}_R(#1)}
\newcommand{\lcm}[1]{\text{lcm}(#1)}
\newcommand{\cl}[1]{\text{cl}(#1)}
%\DeclareMathOperator{\Homm}{\text{Hom}_R( )}
\DeclareMathOperator{\im}{\text{Im}}
\DeclareMathOperator{\Ker}{\text{ker}}
\DeclareMathOperator{\ch}{\text{char}}
\DeclareMathOperator{\spec}{\text{spec}}
\DeclareMathOperator{\mspec}{\text{maxspec}}
\renewcommand{\solutiontitle}{\noindent Ans. \par\noindent}
\begin{document}
\newpage
\tableofcontents
\newpage
\section{The real and complex number system}
Unless the contrary explicitly stated, all numbers that are mentioned in these exercises are understood to be real.
\begin{questions} 
\question If $r$ is rational $(r\neq 0)$ and $x$ is irrational, prove that $r+r$ and $rx$ is irrational.
\begin{solution}
jfgjdfs;gj;jbbnggtggguyy
\end{solution}

\question Prove that there is no rational number whose square is 12.
\begin{solution}

\end{solution}

\question Prove Proposition 1.15\footnote{Proposition 1.15 The axioms for multiplication imply the following statements.\\
(a) If $n\neq 0$ and $xy=xz$ then $y=z.$\\
(b) If $x\neq 0$ and $xy=x$ then $y=1.$\\
(c) If $x\neq 0$ and $xy=1$ then $y=1/x.$\\
(d) If $x\neq 0$ then $1/(1/x)=x.$}


\question Let $E$ be a non empty subset of an ordered set; suppose $\alpha$ is a lower bound of $E$ and $\beta$ is a upper bound of $E$. Prove that $\alpha\leq \beta.$

\question Let $A$ be a non empty set of real numbers which is bounded below. Let $-A$ be the set of all numbers $-x$, where $x\in A.$ Prove that $$\inf A=-\sup(-A).$$

\question Fix $b>1.$
\begin{parts}
\part If $m,n,p,q$ are integers, $n>0,q>0$ and $r=m/n=p/q$, prove that $$(b^m)^{1/q}=(b^p)^{1/q}.$$
Hence it make sense to define $b^r=(b^m)^{1/n}$.
\part Prove that $b^{r+s}=b^rb^s$ if $r$ and $s$ is rational.
\part If $x$ is real, define $B(x)$ to be the set of all numbers $b^t$, where $t$ is rational and $t\leq x$. Prove that $$b^r=\sup B(r)$$ when $r$ is rational. Hence it make sense to define $$b^x=\sup B(x)$$ for every real $x.$
\part Prove that $b^{x+y}=b^xb^y$ for all real $x$ and $y.$
\end{parts}


\question Fix $b>1,y>0$ and prove that there is a unique real $x$ such that $b^x=y$, by completing the following outline. (This $x$ is called the \textit{logarithm} of $y$ to the \textit{base} $b.$)
\begin{parts}
\part For any positive integer $n, b^n-1\geq n(b-1).$
\part Hence $b-1\geq n(b^{1/n}-1).$
\part If $t>1$ and $n>(b-1)/(t-1),$ then $b^{1/n}<t.$
\part If $w$ is such that $b^w<y$ then $b^{w+(1/n)}<y$ for sufficiently large $n;$ to see this, apply part (c) with $t=y\cdot b^{-w}.$
\part If $b^w>y$ then $b^{w-(1/n)}>y$ for sufficiently large $n$.
\part Let $A$ be the set of all $w$ such that $b^w<y$, and show that $x=\sup A$ satisfies $b^x=y.$
\part Prove that this $x$ is unique.
\end{parts}

\question Prove that no order can be defined in the complex field that turns it into an ordered field. Hint: $-1$ is a square.

\question Suppose, $z=a+bi,w=c+di$. Define $z<w$ if $a<c$, and also if $a=c$ but $b<d$. Prove that this turns the set of all complex numbers into an ordered set. (This type of order relation is called a \textit{dictionary order,} or \textit{lexicographic order,} for obvious reason.) Does this ordered set have least upper bound property?

\question Suppose $z=a+bi,w=u+iv$, and \begin{align*}
a=\left(\dfrac{|w|+u}{2}\right)^{1/2},\qquad b=\left(\dfrac{|w|-u}{2}\right)^{1/2}.
\end{align*}
Prove that $z^2=w$ if $v\geq 0$ and $(\bar{z}^2=w$ if $v\leq 0$. Conclude that every complex number (with one exception!) has two complex square roots.

\question If $z$ is a complex number, prove that there is an $r$ and a complex number $w$ with $|w|=1$ such that $z=rw.$ Are $w$ and $r$ is always uniquely determined by $z$?

\question If $z_1,\cdots,z_n$ are complex, prove that $$|z_1+z_2+\cdots+z_n|\leq |z_1|+|z_2|+\cdots+|z_n|.$$

\question If $x,y$ are complex, prove that $$||x|-|y||\leq |x-y|.$$

\question If $z$ is a complex number such that $|z|=1$, that is such that $z\bar{z}=1$, compute $$|1+z|^2+|1-z|^2.$$

\question Under what conditions does the equality hold in the Schwarz inequality?

\question Suppose, $k\geq 3,\textbf{x,y}\in {\RR}^k, |\textbf{x-y}|=d>0,$ and $r>0$. Prove:
\begin{parts}
\part If $2r>d$, there are infinitely many $z\in {\RR}^k$ such that $$|\textbf{z-x}|=|\textbf{z-y}|=r.$$
\part If $2r=d$, there is exactly one such $\textbf{z}.$
\part If $2r<d,$ there is no such $\textbf{z}.$ 
\end{parts} 
How must these statements be modified if $k$ is 2 or 1.

\question Prove that $$|\textbf{x+y}|^2+|\textbf{x-y}|^2=2|\textbf{x}|^2+2|\textbf{y}|^2$$ if $\textbf{x}\in {\RR}^k$ and $\textbf{y}\in {\RR}^k$. Interpret this geometrically, as a statement about parallelograms.

\question If $k\geq 2$ and $\textbf{x}\in {\RR}^k,$ prove that there exists $\textbf{y}\in {\RR}^k$ such that $\textbf{y}\neq \textbf{0}$ but $\textbf{x $\cdot$ y}=\textbf{0}.$ Is this also true for $k=1?$

\question Suppose $\textbf{a}\in {\RR}^k$, $\textbf{b}\in {\RR}^k$. Find $\textbf{c}\in {\RR}^k$ and $r\geq 0$ such that $$|\textbf{x}-\textbf{a}|=2|\textbf{x}-\textbf{b}|$$ if and only if $|\textbf{x}-\textbf{c}|=r.$\\
(\textit{Solution: $3\textbf{c}=4\textbf{b}-\textbf{a},3r=2|\textbf{b}-\textbf{a}|$})

\question With reference to the Appendix, suppose that property $(\mathrm{III})$ were omitted from the definition of a cut. Keep the same definition of order and addition. Show that the resulting ordered set has the least-upper-bound property, that satisfies axiom (A1) to (A4)(with slightly different zero element!) but that (A5) fails.
\end{questions}
\end{document}

相关内容